15
$\begingroup$

Consider the Young diagram of a partition $\lambda = (\lambda_1,\ldots,\lambda_k)$. For a square $(i,j) \in \lambda$, define the hook numbers $h_{(i,j)} = \lambda_i + \lambda_j' -i - j +1$ where $\lambda'$ is the conjugate of $\lambda$.

The hook-length formula shows, in particular, that if $\lambda\vdash n$ then $$\text{$n!\prod_{\square\,\in\,\lambda}\frac1{h_{\square}}$} \qquad \text{is an integer}.$$ Recall the Fibonacci numbers $F(0)=0, \, F(1)=1$ with $F(n)=F(n-1)+F(n-2)$. Define $[0]!_F=1$ and $[n]!_F=F(1)\cdot F(2)\cdots F(n)$ for $n\geq1$.

QUESTION. Is it true that $$\text{$[n]!_F\prod_{\square\,\in\,\lambda}\frac1{F(h_{\square})}$} \qquad \text{is an integer}?$$

$\endgroup$
14
  • 5
    $\begingroup$ More generally, this interesting question can be asked of any strong divisibility sequence instead of the Fibonacci sequence. But let's perhaps not abuse the notation $F\left(n\right)!$ for something that's not the factorial of $F\left(n\right)$. $\endgroup$ Apr 1, 2019 at 3:50
  • 1
    $\begingroup$ Maybe call it $F!(n)$ instead of $F(n)!$. How far has this been checked? $\endgroup$ Apr 1, 2019 at 3:51
  • 9
    $\begingroup$ Maybe this expression can be obtained by a clever substitution of the $q$-hook length formula? $\endgroup$ Apr 1, 2019 at 4:30
  • 1
    $\begingroup$ @darijgrinberg what is a strong divisibility sequence? Product of any $k$ consecutive guys is divisibly by the product of first $k$ guys? $\endgroup$ Apr 1, 2019 at 8:29
  • 5
    $\begingroup$ For searching purposes: the product of consecutive Fibonacci numbers is sometimes referred to as a fibonorial. $\endgroup$ Apr 1, 2019 at 10:52

2 Answers 2

10
$\begingroup$

Sam is correct of course about $q$-hook formula. Below is a short self-contained proof not relying on such advanced combinatorics.

Denote $h_1>\ldots>h_k$ the set of hook lengths of the first column of diagram $\lambda$. Then the multiset of hooks is $\cup_{i=1}^k \{1,2,\ldots,h_i\}\setminus \{h_i-h_j:i<j\}$ and $n=\sum_i h_i-\frac{k(k-1)}2$.

Recall that $F(m)=P_m(\alpha,\beta)=\prod_{d|m,d>1}\Phi_d(\alpha,\beta)=\prod_d (\Phi_d(\alpha,\beta))^{\eta_d(m)}$, where

$\alpha,\beta=(1\pm \sqrt{5})/2$;

$P_n(x,y)=x^{n-1}+x^{n-2}y+\ldots+y^{n-1}$;

$\Phi_d$ are homogeneous cyclotomic polynomials;

$\eta_d(m)=\chi_{\mathbb{Z}}(m/d)$ (i.e., it equals to 1 if $d$ divides $m$, and to 0 otherwise).

Therefore it suffices to prove that for any fixed $d>1$ we have $$ \sum_{m=1}^n \eta_d(m)+\sum_{i<j}\eta_d(h_i-h_j)-\sum_{i=1}^k\sum_{j=1}^{h_i}\eta_d(j)\geqslant 0.\quad (\ast) $$ $(\ast)$ rewrites as $$ [n/d]+|i<j:h_i\equiv h_j \pmod d|-\sum_{i=1}^k [h_i/d]\geqslant 0.\quad (\bullet) $$ LHS of $(\bullet)$ does not change if we reduce all $h_i$'s modulo $d$ (and accordingly change $n=\sum_i h_i-\frac{k(k-1)}2$, of course), so we may suppose that $0\leqslant h_i\leqslant d-1$ for all $i$. For $a=0,1,\dots, d-1$ denote $t_a=|i:h_i=a|$. Then $(\bullet)$ rewrites as $$ \left[\frac{-\binom{\sum_{i=0}^{d-1} t_i}2+\sum_{i=0}^{d-1} it_i}d\right]+ \sum_{i=0}^{d-1} \binom{t_i}2\geqslant 0. \quad (\star) $$

It remains to observe that LHS of $(\star)$ equals to $$ \left[\frac1d\sum_{i<j}\binom{t_i-t_j}2 \right]. $$

$\endgroup$
4
  • $\begingroup$ Nice. So what does this integer count? $\endgroup$ Apr 1, 2019 at 19:47
  • $\begingroup$ Yes, that was my plan to ask next. $\endgroup$ Apr 1, 2019 at 19:53
  • $\begingroup$ @Fedor: what are $\alpha$ and $\beta$? What's the connection between $P_m$ and $\eta_d$, etc? $\endgroup$ Apr 2, 2019 at 18:36
  • $\begingroup$ @T.Amdeberhan sorry, forgot to copy the notations from my previous Fibonacci answer. Hope now it is clear. $\endgroup$ Apr 2, 2019 at 18:58
6
$\begingroup$

This is a less elementary but maybe more conceptual proof, also giving some combinatorial meaning: Use the formulas $F(n) = \frac{\varphi^n -\psi^n}{\sqrt{5}}$, $\varphi =\frac{1+\sqrt{5}}{2}, \psi = \frac{1-\sqrt{5}}{2}$. Let $q=\frac{\psi}{\varphi} = \frac{\sqrt{5}-3}{2}$, so that $F(n) = \frac{\varphi^n}{\sqrt{5}} (1-q^n)$

Then the Fibonacci hook-length formula becomes:

\begin{align*} f^{\lambda}_F:= \frac{[n]!_F}{\prod_{u\in \lambda}F(h(u))} = \frac{ \varphi^{ \binom{n+1}{2} } [n]!_q }{ \varphi^{\sum_{u \in \lambda} h(u)} \prod_{u \in \lambda} (1-q^{h(u)})} \end{align*} So we have an ordinary $q$-analogue of the hook-length formula. Note that $$\sum_{u \in \lambda} h(u) = \sum_{i} \binom{\lambda_i}{2} + \binom{\lambda'_j}{2} + |\lambda| = b(\lambda) +b(\lambda') +n$$ Using the $q-$analogue hook-length formula via major index (EC2, Chapter 21) we have

\begin{align*} f^\lambda_F = \varphi^{ \binom{n}{2} -b(\lambda)-b(\lambda')} q^{-b(\lambda)} \sum_{T\in SYT(\lambda)} q^{maj(T)} = (-q)^{\frac12( -\binom{n}{2} +b(\lambda') -b(\lambda))}\sum_T q^{maj(T)} \end{align*}

Now, it is clear from the q-HLF formula that $q^{maj(T)}$ is a symmetric polynomial, with lowest degree term $b(\lambda)$ and maximal degree $b(\lambda) + \binom{n+1}{2} - n -b(\lambda) -b(\lambda') =\binom{n}{2} - b(\lambda')$ so the median degree term is $$M=\frac12 \left(b(\lambda) +\binom{n}{2} - b(\lambda')\right)$$ which cancels with the factor of $q$ in $f^{\lambda}_F$, so the resulting polynomial is of the form \begin{align*} f^{\lambda}_F = (-1)^{M} \sum_{T: maj(T) \leq M } (q^{M-maj(T)} + q^{maj(T)-M}) \\ = (-1)^{M} \sum_{T} (-1)^{M-maj(T)}( \varphi^{2(M-maj(T))} + \psi^{2(M-maj(T)}) = \sum_T (-1)^{maj(T)} L(2(M-maj(T))) \end{align*} where $L$ are the Lucas numbers.

**byproduct of collaborations with A. Morales and I. Pak.

$\endgroup$
2
  • $\begingroup$ Nice, thank you. This would be even more fitting to the 2nd part of this question mathoverflow.net/questions/327015/… Therefore, do you like to post it there as well? $\endgroup$ Apr 3, 2019 at 15:50
  • $\begingroup$ Thanks! I just pasted it there. $\endgroup$ Apr 3, 2019 at 17:55

Your Answer

By clicking “Post Your Answer”, you agree to our terms of service and acknowledge you have read our privacy policy.

Not the answer you're looking for? Browse other questions tagged or ask your own question.